- Sat Aug 19, 2017 1:00 pm
#38549
Complete Question Explanation
(The complete setup for this game can be found here: lsat/viewtopic.php?t=15133)
The correct answer choice is (B)
This is the infamous Rule Substitution question, a staple in the Logic Games section as of late. Considering their growing popularity, tackling such questions is no longer merely optional: it is imperative to have a workable strategy in place. Although the difficulty of this question may have been compounded by the fact that it was the final question in the section, a closer look reveals that the correct answer choice (B) is a straight restatement of the rule in question. Do not assume that the Rule Substitution question will necessarily be difficult! If you skip such questions by default, you may be robbing yourself of an easy point.
The question stem asks you to replace the fourth rule of the game, according to which if M is published in the fall, N must be published in the spring:
Since Additional Effects are easier to spot, look for answer choices that contain such conditions first. Answer choice (A), for instance, addresses the placement of L, which was never addressed in the original rule set. Answer choice (A) can be eliminated for that reason alone. Likewise, answer choice (D) contains a rule whose contrapositive contradicts the rule being replaced:
We now need to consider the remaining three answer choices more closely:
Answer choice (B) is the correct answer choice. This rule establishes the following conditional relationship:
The better question is: does answer choice (B) contain a Partial Match? It does not, because M and P cannot be assigned to the same group as each other (first rule), and so P being published in the fall is synonymous with saying that M is published in the spring. Thus, the effect of this rule is identical to that of the contrapositive of the original rule:
Answer choice (E) establishes the following conditional relationship:
This rule is consistent with our original inference that if N is published in the fall, then so is O (see conditional chain). Answer choice (E), therefore, does not contain an Additional Effects rule. However, it does contain a Partial Match, as it does not force a relationship between M and N identical to that of the original rule. Nothing is stopping us, for instance, from publishing both M and N in the fall, along with K and O. Such a solution would be in violation of the original rule set, but nevertheless consistent with the rule in answer choice (E):
Since answer choice (E) allows for a solution prohibited by the original set of rules, it contains a Partial Match and is therefore incorrect.
(The complete setup for this game can be found here: lsat/viewtopic.php?t=15133)
The correct answer choice is (B)
This is the infamous Rule Substitution question, a staple in the Logic Games section as of late. Considering their growing popularity, tackling such questions is no longer merely optional: it is imperative to have a workable strategy in place. Although the difficulty of this question may have been compounded by the fact that it was the final question in the section, a closer look reveals that the correct answer choice (B) is a straight restatement of the rule in question. Do not assume that the Rule Substitution question will necessarily be difficult! If you skip such questions by default, you may be robbing yourself of an easy point.
The question stem asks you to replace the fourth rule of the game, according to which if M is published in the fall, N must be published in the spring:
- MF NS
Contrapositive: NF MS
Since Additional Effects are easier to spot, look for answer choices that contain such conditions first. Answer choice (A), for instance, addresses the placement of L, which was never addressed in the original rule set. Answer choice (A) can be eliminated for that reason alone. Likewise, answer choice (D) contains a rule whose contrapositive contradicts the rule being replaced:
- Answer choice (D): NS MS
Contrapositive: MF NF
We now need to consider the remaining three answer choices more closely:
Answer choice (B) is the correct answer choice. This rule establishes the following conditional relationship:
- NF PF
The better question is: does answer choice (B) contain a Partial Match? It does not, because M and P cannot be assigned to the same group as each other (first rule), and so P being published in the fall is synonymous with saying that M is published in the spring. Thus, the effect of this rule is identical to that of the contrapositive of the original rule:
- NF PF MS
- MS PF
Contrapositive: PS MF
Answer choice (E) establishes the following conditional relationship:
- OS .....NS
Contrapositive: NF OF
This rule is consistent with our original inference that if N is published in the fall, then so is O (see conditional chain). Answer choice (E), therefore, does not contain an Additional Effects rule. However, it does contain a Partial Match, as it does not force a relationship between M and N identical to that of the original rule. Nothing is stopping us, for instance, from publishing both M and N in the fall, along with K and O. Such a solution would be in violation of the original rule set, but nevertheless consistent with the rule in answer choice (E):
Since answer choice (E) allows for a solution prohibited by the original set of rules, it contains a Partial Match and is therefore incorrect.
You do not have the required permissions to view the files attached to this post.